Parte 2: Examen Simulador de Cálculo UNAM | Áreas 1 y 2

¡Falicidades por llegar hasta acá aspirante! Vamos con la solución de la segunda y última parte del examen simulador de cálculo para el examen de admisión a la UNAM.

SIMULACRO-UNAM-CALCULO

Este simulador tiene ejercicios similares al examen para que puedas practicar todos los temas de cálculo. Resuélvelos por tu cuenta antes de mirar la solución.

Aparta tu lugar al primer grupo con 25% OFF

Curso UNAM 2025

Acceso total durante x12 meses
El curso cubre todos los temas del examen de admisión con clases en vivo y examenes simulacro.
Código: UNAM120
$4,000 MX
$3,000 MXN

Simulador de Matemáticas UNAM

Continuamos resolviendo los últimos 10 ejercicios de cálculo en el simulador real para la UNAM. Recuerda tomar descansos entre series de ejercicios, esto también forma parte del proceso de aprendizaje.

Reactivo 11

Calcule la derivada de la función g\left(t\right)=\frac{1+t}{\sqrt{1-t}} , empleando la regla de la cadena.

  1. \frac{3-t}{2{\left(-t\right)}^{\frac{3}{2}}}
  2. \frac{3-t}{2{\left(1-t\right)}^{\frac{3}{2}}}
  3. \frac{3-t}{2{\left(t\right)}^{\frac{3}{2}}}
  4. \frac{3-t}{2{\left(t-1\right)}^{\frac{3}{2}}}

Solución:

Según la regla de la cadena, en este caso debemos comenzar derivando a la función racional, ya que es ella la que contiene al binomio del numerador y a la función radical del denominador.

{g}^{\text{'}}\left(t\right)=\frac{{\left(1+t\right)}^{\text{'}}\left(\sqrt{1-t}\right)-{\left(\sqrt{1-t}\right)}^{\text{'}}\left(1+t\right)}{{\left(\sqrt{1-t}\right)}^{2}}

Resolvemos las derivadas indicadas. Aplicamos la derivada de una suma y la derivada de un radical respectivamente.

{g}^{\text{'}}\left(t\right)=\frac{\left(1\right)\left(\sqrt{1-t}\right)-\left[\frac{1}{2}{\left(1-t\right)}^{\frac{1}{2}-1}{\left(1-t\right)}^{\text{'}}\right]\left(1+t\right)}{1-t}=\frac{\sqrt{1-t}-\left[\frac{1}{2}{\left(1-t\right)}^{-\frac{1}{2}}\left(-1\right)\right]\left(1+t\right)}{1-t}

Simplificamos.

=\frac{\sqrt{1-t}-\left[-\frac{1}{2}\frac{1}{\sqrt{1-t}}\right]\left(1+t\right)}{1-t}=\frac{\sqrt{1-t}+\frac{1}{2}\frac{1+t}{\sqrt{1-t}}}{1-t}=\frac{2{\left(\sqrt{1-t}\right)}^{2}+\left(1+t\right)}{2\left(1-t\right)\sqrt{1-t}}

=\frac{2\left(1-t\right)+1+t}{2{\left(1-t\right)}^{\frac{3}{2}}}=\frac{2+1-2t+t}{2{\left(1-t\right)}^{\frac{3}{2}}}=\frac{3-t}{2{\left(1-t\right)}^{\frac{3}{2}}}

Finalmente:

{g}^{\text{'}}\left(x\right)=\frac{3-t}{2{\left(1-t\right)}^{\frac{3}{2}}}

La respuesta correcta es el inciso b).

Reactivo 12

Determine la tercera derivada para la función y=4{x}^{4}+2x-1 .

  1. {y}^{\text{'}\text{'}\text{'}}=-96x
  2. {y}^{\text{'}\text{'}\text{'}}=16{x}^{3}+2
  3. {y}^{\text{'}\text{'}\text{'}}=96x
  4. {y}^{\text{'}\text{'}\text{'}}=48{x}^{2}

Solución:

Las derivadas de orden superior, se obtienen derivando sucesivamente a una función, tantas veces como se indique. En este caso, el enunciado solicita la tercera derivada, por tanto, debemos derivar a y 3 veces consecutivas.

Primera derivada.

{y}^{\text{'}}={\left(4{x}^{4}+2x-1\right)}^{\text{'}}=4\cdot 4{x}^{3}+2=16{x}^{3}+2

Segunda derivada.

{y}^{\text{'}\text{'}}={\left({y}^{\text{'}}\right)}^{\text{'}}={\left(16{x}^{3}+2\right)}^{\text{'}}=48{x}^{2}

Tercera derivada.

{y}^{\text{'}\text{'}\text{'}}={\left({y}^{\text{'}\text{'}}\right)}^{\text{'}}={\left(48{x}^{2}\right)}^{\text{'}}=96x

La tercera derivada de y=4{x}^{4}+2x-1 es:

{y}^{\text{'}\text{'}\text{'}}=96x

Concluimos indicando la opción c) como respuesta correcta.

Reactivo 13

La trayectoria de una pelota luego de ser pateada, puede describirse a través de la función s\left(t\right)=-\frac{{t}^{2}}{10}+t . Calcule la rapidez en t=4 .

  1. 0.8\frac{\mathrm{m}}{\mathrm{s}}
  2. 0.2\frac{\mathrm{m}}{\mathrm{s}}
  3. -0.2\frac{\mathrm{m}}{\mathrm{s}}
  4. 0.1\frac{\mathrm{m}}{\mathrm{s}}

Solución:

Según las relaciones elementales de la cinemática, el desplazamiento de un cuerpo y su velocidad en cada uno de esos puntos, viene dada por:

v=\frac{ds\left(t\right)}{dt}

Es decir, la función rapidez es igual a la derivada respecto del tiempo del desplazamiento. Procedemos a derivar a s :

v\left(t\right)={\left(-\frac{{t}^{2}}{10}+t\right)}^{\text{'}}=-\frac{2}{10}t+1

Ahora sustituimos t=4 .

v\left(t=4\right)=-\frac{2}{10}\left(4\right)+1=0.2 \mathrm{m}/\mathrm{s}

La rapidez del cuerpo para t=4 \mathrm{s} es de 0.2 metros por segundo.

Concluimos seleccionando al inciso b) como la respuesta correcta.

Reactivo 14

Calcule el resultado de la siguiente integral inmediata.

\int \frac{3}{4}{x}^{2} dx

  1. \frac{{x}^{2}}{4}+C
  2. -\frac{{x}^{3}}{4}+C
  3. \frac{{x}^{3}}{4}+C
  4. \frac{{x}^{3}}{3}+C

Solución:

Antes de aplicar la fórmula para la integral de una potencia, debemos extraer la constante del integrando aplicando la propiedad de una constante por una función.

\int \frac{3}{4}{x}^{2} dx=\frac{3}{4}\int {x}^{2} dx

Aplicando la fórmula:

\int {x}^{n}dx=\frac{{x}^{n+1}}{n+1}+C

\frac{3}{4}\int {x}^{2} dx=\frac{3}{4}\cdot \frac{{x}^{3}}{3}+C=\frac{{x}^{3}}{4}+C

Finalmente:

\int \frac{3}{4}{x}^{2} dx=\frac{{x}^{3}}{4}+C

La respuesta correcta es el inciso c).

Reactivo 15

Calcule la siguiente integral indefinida.

\int \frac{\sqrt[3]{x}-x\sqrt{x}}{{x}^{2}}dx

  1. -\frac{3}{2}{x}^{-\frac{2}{3}}+2{x}^{\frac{1}{2}}+C
  2. -\frac{3}{2}{x}^{-\frac{2}{3}}-2{x}^{\frac{1}{2}}+C
  3. -\frac{3}{2}{x}^{-\frac{2}{3}}+2{x}^{-\frac{1}{2}}+C
  4. \frac{3}{2}{x}^{-\frac{2}{3}}-2{x}^{-\frac{1}{2}}+C

Solución:

Comenzamos por separar a la fracción, para simplificar la solución.

\int \frac{\sqrt[3]{x}-x\sqrt{x}}{{x}^{2}}dx=\int \frac{\sqrt[3]{x}}{{x}^{2}}-\frac{x\sqrt{x}}{{x}^{2}} dx

Ahora simplificamos las fracciones.

\int \frac{\sqrt[3]{x}}{{x}^{2}}-\frac{x\sqrt{x}}{{x}^{2}} dx=\int {x}^{-\frac{5}{3}}-{x}^{-\frac{1}{2}} dx

Separamos la integral en dos y aplicamos la integral de una potencia a cada una.

\int {x}^{-\frac{5}{3}}-{x}^{-\frac{1}{2}} dx=\int {x}^{-\frac{5}{3}} dx-\int {x}^{-\frac{1}{2}}dx

\int {x}^{-\frac{5}{3}} dx-\int {x}^{-\frac{1}{2}}dx=\frac{{x}^{-\frac{5}{3}+1}}{-\frac{5}{3}+1}-\frac{{x}^{-\frac{1}{2}+1}}{-\frac{1}{2}+1}+C

=\frac{{x}^{-\frac{2}{3}}}{-\frac{2}{3}}-\frac{{x}^{\frac{1}{2}}}{\frac{1}{2}}+C=-\frac{3}{2}{x}^{-\frac{2}{3}}-2{x}^{\frac{1}{2}}+C

Finalmente:

\int \frac{\sqrt[3]{x}-x\sqrt{x}}{{x}^{2}}dx=-\frac{3}{2}{x}^{-\frac{2}{3}}-2{x}^{\frac{1}{2}}+C

Concluimos que la respuesta correcta es el inciso b).

Reactivo 16

Obtenga el resultado de la siguiente integral trigonométrica.

\int \frac{1}{1-\mathrm{sin}x}dx

  1. \mathrm{cot}x+\mathrm{csc}x+C
  2. -\mathrm{tan}x+\mathrm{sec}x+C
  3. \mathrm{tan}x+\mathrm{sec}x+C
  4. \mathrm{tan}x-\mathrm{sec}x+C

Solución:

Para este integrando no existe ninguna fórmula inmediata de integración y tampoco podemos separar la fracción, por tanto, debemos manipular algebraicamente la fracción. Comencemos multiplicando y dividiendo por el conjugado del denominador.

\int \frac{1+\mathrm{sin}x}{\left(1-\mathrm{sin}x\right)\left(1+\mathrm{sin}x\right)}dx=\int \frac{1+\mathrm{sin}x}{1-{\mathrm{sin}}^{2}x}dx

Transformamos 1-{\mathrm{sin}}^{2}x en {\mathrm{cos}}^{2}x aplicando la identidad pitagórica.

\int \frac{1+\mathrm{sin}x}{1-{\mathrm{sin}}^{2}x}dx=\int \frac{1+\mathrm{sin}x}{{\mathrm{cos}}^{2}x}dx=\int \left(\frac{1}{{\mathrm{cos}}^{2}x}+\frac{\mathrm{sin}x}{{\mathrm{cos}}^{2}x}\right)dx

Separamos las integrales.

=\int {\mathrm{sec}}^{2}xdx+\int \mathrm{tan}x\mathrm{sec}xdx

Para estas dos integrales si existen fórmulas inmediatas. Procedemos a la solución.

\int {\mathrm{sec}}^{2}xdx+\int \mathrm{tan}x\mathrm{sec}xdx=\mathrm{tan}x+\mathrm{sec}x+C

Finalmente:

\int \frac{1}{1-\mathrm{sin}x}dx=\mathrm{tan}x+\mathrm{sec}x+C

La respuesta correcta es la c).

Reactivo 17

Calcule la integral indefinida, empleando el método de cambio de variable.

\int \frac{x}{{x}^{2}+1}\mathrm{ln}\left({x}^{2}+1\right)dx

  1. \mathrm{ln}\left({x}^{2}+1\right)+C
  2. \frac{{\left[\mathrm{ln}\left({x}^{2}+1\right)\right]}^{-2}}{4}+C
  3. \frac{{\left[\mathrm{ln}\left({x}^{2}-1\right)\right]}^{2}}{4}+C
  4. \frac{{\left[\mathrm{ln}\left({x}^{2}+1\right)\right]}^{2}}{4}+C

Solución:

En el método de integración por cambio de variables, tenemos que identificar en el integrando el producto de una función f\left(x\right) y a su derivada {f}^{\text{'}}\left(x\right) . Claramente f\left(x\right) sería \mathrm{ln}\left({x}^{2}+1\right) y su derivada es \frac{2x}{{x}^{2}+1} . Esta se encuentra en el integrando, pero le falta el dos.

Multiplicamos y dividimos por 2.

\int \frac{x}{{x}^{2}+1}\mathrm{ln}\left({x}^{2}+1\right)dx=\frac{1}{2}\int \frac{2x}{{x}^{2}+1}\mathrm{ln}\left({x}^{2}+1\right)dx

Aplicando cambio de variables:

u=\mathrm{ln}\left({x}^{2}+1\right)\to du=\frac{2x}{{x}^{2}+1}dx

Sustituyendo nos queda:

\frac{1}{2}\int u du

Resolvemos aplicando la fórmula de la integral de una potencia.

\frac{1}{2}\int u du=\frac{1}{2}\frac{{u}^{2}}{2}+C

Devolvemos el cambio de variables.

\frac{1}{2}\frac{{u}^{2}}{2}+C\to \frac{{\left[\mathrm{ln}\left({x}^{2}+1\right)\right]}^{2}}{4}+C

Finalmente:

\int \frac{x}{{x}^{2}+1}\mathrm{ln}\left({x}^{2}+1\right)dx=\frac{{\left[\mathrm{ln}\left({x}^{2}+1\right)\right]}^{2}}{4}+C

Seleccionamos como respuesta correcta la d).

Reactivo 18

Determine el resultado de la siguiente integral indefinida aplicando el método por partes.

\int {x}^{2}\mathrm{ln}\left(x+1\right)dx

  1. \frac{{x}^{3}\mathrm{ln}{\left(x+1\right)}^{2}}{3}-\frac{{x}^{3}}{9}+\frac{{x}^{2}}{6}-\frac{x}{3}+\frac{1}{3}\mathrm{ln}{\left(x+1\right)}^{2}+C
  2. \frac{{x}^{3}\mathrm{ln}\left(x-1\right)}{3}-\frac{{x}^{3}}{9}+\frac{{x}^{2}}{6}-\frac{x}{3}+\frac{1}{3}\mathrm{ln}\left(x-1\right)+C
  3. \frac{{x}^{3}\mathrm{ln}\left(x+1\right)}{3}-\frac{{x}^{3}}{9}+\frac{{x}^{2}}{6}-\frac{x}{3}+\frac{1}{3}\mathrm{ln}\left(x+1\right)+C
  4. \frac{{x}^{3}\mathrm{ln}\left(x+1\right)}{3}-\frac{2{x}^{3}}{9}+\frac{{5x}^{2}}{6}-\frac{x}{3}-\frac{1}{3}\mathrm{ln}\left(x+1\right)+C

Solución:

En la integración por partes, debemos identificar en el integrando quien debe ser u y dv . Uno de los objetivos principales de este método, es eliminar o simplificar funciones especiales como logaritmos y trigonométricas.

El integrando está conformado por una función polinomial y una logarítmica. Para deshacer a la logarítmica, es necesario derivar. La selección de u y dv quedaría:

u=\mathrm{ln}\left(x+1\right), dv={x}^{2}

Derivando e integrando:

du=\frac{1}{x+1}dx, v=\frac{{x}^{3}}{3}

Aplicando la definición de integración por partes:

\int udv=uv-\int vdu

Sustituyendo:

I=\frac{{x}^{3}\mathrm{ln}\left(x+1\right)}{3}-\frac{1}{3}\int \frac{{x}^{3}}{x+1}dx

Llamamos al término integral {I}_{2} .

{I}_{2}=\frac{1}{3}\int \frac{{x}^{3}}{x+1}dx

La vía más rápida será aplicar división de polinomios en el integrando.

{x}^{3}÷\left(x+1\right)={x}^{2}-x+1-\frac{1}{x+1}

Sustituimos en la integral y separamos.

{I}_{2}=\frac{1}{3}\int \left({x}^{2}-x+1-\frac{1}{x+1}\right)dx=\frac{1}{3}\left[\int \left({x}^{2}-x+1\right)dx-\int \frac{1}{x+1}dx\right]

Resolvemos aplicando la integral de una potencia y la integral del inverso de x .

=\frac{1}{3}\left[\frac{{x}^{3}}{3}-\frac{{x}^{2}}{2}+x-\mathrm{ln}\left(x+1\right)\right]+{c}_{1}=\frac{{x}^{3}}{9}-\frac{{x}^{2}}{6}+\frac{x}{3}-\frac{1}{3}\mathrm{ln}\left(x+1\right)+{c}_{1}

Sustituimos en la expresión original.

I=\frac{{x}^{3}\mathrm{ln}\left(x+1\right)}{3}-\left[\frac{{x}^{3}}{9}-\frac{{x}^{2}}{6}+\frac{x}{3}-\frac{1}{3}\mathrm{ln}\left(x+1\right)\right]+C

=\frac{{x}^{3}\mathrm{ln}\left(x+1\right)}{3}-\frac{{x}^{3}}{9}+\frac{{x}^{2}}{6}-\frac{x}{3}+\frac{1}{3}\mathrm{ln}\left(x+1\right)+C

Finalmente:

\int {x}^{2}\mathrm{ln}\left(x+1\right)dx=\frac{{x}^{3}\mathrm{ln}\left(x+1\right)}{3}-\frac{{x}^{3}}{9}+\frac{{x}^{2}}{6}-\frac{x}{3}+\frac{1}{3}\mathrm{ln}\left(x+1\right)+C

Comparando con los incisos, concluimos que la respuesta correcta es c).

Reactivo 19

Calcule el resultado de la siguiente integral definida.

{\int }_{2}^{3}\frac{x}{{x}^{2}-1}dx

  1. \frac{1}{2}\left[\mathrm{ln}\left(8\right)-\mathrm{ln}\left(3\right)\right]
  2. \frac{1}{2}\left[\mathrm{ln}\left(8\right)+\mathrm{ln}\left(3\right)\right]
  3. -\frac{1}{2}\left[\mathrm{ln}\left(8\right)-\mathrm{ln}\left(3\right)\right]
  4. \frac{1}{2}\left[\mathrm{ln}\left(3\right)-\mathrm{ln}\left(8\right)\right]

Solución:

Para calcular una integral definida, aplicamos el segundo teorema fundamental del cálculo.

{\int }_{a}^{b}f\left(x\right)dx=F\left(b\right)-F\left(a\right)

Es decir, integramos indefinidamente y luego evaluamos a la primitiva en los extremos. Comenzamos aplicando cambio de variables.

u={x}^{2}-1\to du=2xdx\to xdx=\frac{1}{2}du

En este caso, los extremos de integración también cambian.

a\to {u}_{a}, b\to {u}_{b}

\frac{1}{2}{\int }_{{u}_{a}}^{{u}_{b}}\frac{du}{u}=\left.\begin{array}{c} \\ \frac{1}{2}\mathrm{ln}u\\  \end{array}\right]\begin{array}{c}{u}_{b}\\ {u}_{a}\end{array}

Devolvemos el cambio de variables y evaluamos.

\left.\begin{array}{c} \\ \frac{1}{2}\mathrm{ln}\left({x}^{2}-1\right)\\  \end{array}\right]\begin{array}{c}3\\ 2\end{array}=\frac{1}{2}\left[\mathrm{ln}\left({3}^{2}-1\right)-\mathrm{ln}\left({2}^{2}-1\right)\right]=\frac{1}{2}\left[\mathrm{ln}\left(8\right)-\mathrm{ln}\left(3\right)\right]

Finalmente:

{\int }_{2}^{3}\frac{x}{{x}^{2}-1}dx=\frac{1}{2}\left[\mathrm{ln}\left(8\right)-\mathrm{ln}\left(3\right)\right]

La respuesta correcta es la a).

Reactivo 20

¿Qué método de integración utilizaría para resolver la siguiente integral?

\int \frac{8{x}^{2}+1}{{x}^{3}+2{x}^{2}-x+1}dx

  1. Integración directa
  2. Cambio de variables
  3. Por partes
  4. Fracciones parciales

Solución:

Parece una pregunta simple, pero cuando nos enfrentamos a integrales más complejas, comúnmente es necesario aplicar más de un método de integración, razón por la que es indispensable determinar por cuál es más conveniente empezar.

A simple vista podemos determinar que no es factible un cambio de variables, ya que ningún polinomio es la derivada del otro. La integración por partes también queda descartada, ya que integrar o derivar \frac{1}{{x}^{3}+2{x}^{2}-x+1} terminaría en una integral más compleja.

La integración directa tampoco es una opción, porque no hay forma de aplicar artificios matemáticos para que se pueda emplear la integración directa. Además, el numerador tiene menor grado que el denominador, por lo que no es posible aplicar la división de polinomios.

En vista de todo lo anterior, el método que debemos emplear es el de fracciones parciales. Se descompone a la fracción en fracciones con polinomios en el denominador de menor grado.

Concluimos indicando que la respuesta correcta es el inciso d).